Physical Assessment Exam 2

अब Quizwiz के साथ अपने होमवर्क और परीक्षाओं को एस करें!

The nurse is assessing the skin of a patient who has acquired immunodeficiency syndrome (AIDS) and notices multiple patchlike lesions on the temple and beard area that are faint pink in color. The nurse recognizes these lesions as:

* Answer: Kaposis sarcoma. Rationale: Kaposis sarcoma is a vascular tumor that, in the early stages, appears as multiple, patchlike, faint pink lesions over the patients temple and beard areas. Measles is characterized by a red-purple maculopapular blotchy rash that appears on the third or fourth day of illness. The rash is first observed behind the ears, spreads over the face, and then spreads over the neck, trunk, arms, and legs. Cherry (senile) angiomas are small (1 to 5 mm), smooth, slightly raised bright red dots that commonly appear on the trunk in all adults over 30 years old. Herpes zoster causes vesicles up to 1 cm in size that are elevated with a cavity containing clear fluid.

A patient has been shown to have a sensorineural hearing loss. During the assessment, it would be important for the nurse to:

. Answer: Ask the patient what medications he is currently taking. Rationale: A simple increase in amplitude may not enable the person to understand spoken words. Sensorineural hearing loss may be caused by presbycusis, which is a gradual nerve degeneration that occurs with aging and by ototoxic drugs, which affect the hair cells in the cochlea.

During an interview, the patient states he has the sensation that everything around him is spinning. The nurse recognizes that the portion of the ear responsible for this sensation is the:

. Answer: Labyrinth. Rationale: If the labyrinth ever becomes inflamed, then it feeds the wrong information to the brain, creating a staggering gait and a strong, spinning, whirling sensation called vertigo.

The nurse recognizes that this symptom indicates:

. Answer: Tinnitus. Rationale: Tinnitus is a sound that comes from within a person; it can be a ringing, crackling, or buzzing sound. It accompanies some hearing or ear disorders.

When examining the eye, the nurse notices that the patients eyelid margins approximate completely. The nurse recognizes that this assessment finding:

Answer: Is expected. Rationale: The palpebral fissure is the elliptical open space between the eyelids, and, when closed, the lid margins approximate completely, which is a normal finding.

The nurse has discovered decreased skin turgor in a patient and knows that this finding is expected in which condition?

Answer: Severe dehydration Rationale: Decreased skin turgor is associated with severe dehydration or extreme weight loss.

During an assessment, a patient mentions that I just cant smell like I used to. I can barely smell the roses in my garden. Why is that? For which possible causes of changes in the sense of smell will the nurse assess? Select all that apply.

Answer: -Cigarette smoking -Chronic allergies -Aging

A patient has been admitted to a hospital after the staff in the nursing home noticed a pressure ulcer in his sacral area. The nurse examines the pressure ulcer and determines that it is a stage II ulcer. Which of these findings are characteristic of a stage II pressure ulcer? Select all that apply.

Answer: -Partial thickness skin erosion is observed with a loss of epidermis or dermis. -Open blister areas have a red-pink wound bed. Rationale: Stage I pressure ulcers have intact skin that appears red but is not broken, and localized redness in intact skin will blanche with fingertip pressure. Stage II pressure ulcers have partial thickness skin erosion with a loss of epidermis or also the dermis; open blisters have a red-pink wound bed. Stage III pressure ulcers are full thickness, extending into the subcutaneous tissue; subcutaneous fat may be seen but not muscle, bone, or tendon. Stage IV pressure ulcers involve all skin layers and extend into supporting tissue, exposing muscle, bone, and tendon. Slough (stringy matter attached to the wound bed) or eschar (black or brown necrotic tissue) may be present.

During an examination, a patient states that she was diagnosed with open-angle glaucoma 2 years ago. The nurse assesses for characteristics of open-angle glaucoma. Which of these are characteristics of open-angle glaucoma?Select all that apply.

Answer: -Patient experiences tunnel vision in the late stages. -Vision loss begins with peripheral vision. -Virtually no symptoms are exhibited. Rationale: Open-angle glaucoma is the most common type of glaucoma; virtually no symptoms are exhibited. Vision loss begins with the peripheral vision, which often goes unnoticed because individuals learn to compensate intuitively by turning their heads. The other characteristics are those of closed-angle glaucoma.

A patient with a known history of heavy alcohol use has been admitted to the ICU after he was found unconscious outside a bar. The nurse closely monitors him for symptoms of withdrawal. Which of these symptoms may occur during this time? Select all that apply.

Answer: Coarse tremor of the hands Transient hallucinations Sweating Rationale: Symptoms of uncomplicated alcohol withdrawal start shortly after the cessation of drinking, peak at the second day, and improve by the fourth or fifth day. Symptoms include coarse tremors of the hands, tongue, and eyelids; anorexia; nausea and vomiting; autonomic hyperactivity (e.g., tachycardia, sweating, elevated blood pressure); and transient hallucinations, among other symptoms.

A patient visits the clinic to ask about smoking cessation. He has smoked heavily for 30 years and wants to stop cold turkey. He asks the nurse, What symptoms can I expect if I do this? Which of these symptoms should the nurse share with the patient as possible symptoms of nicotine withdrawal? Select all that apply.

Answer: Headaches Hunger Restlessness Nervousness Rationale: Symptoms of nicotine withdrawal include vasodilation, headaches, anger, irritability, frustration, anxiety, nervousness, awakening at night, difficulty concentrating, depression, hunger, impatience, and the desire to smoke.

The nurse is testing the hearing of a 78-year-old man and is reminded of the changes in hearing that occur with aging that include which of the following? Select all that apply.

Answer: -Progression of hearing loss is slow. -The aging person may find it harder to hear consonants than vowels. -Sounds may be garbled and difficult to localize. Rationale: Presbycusis is a type of hearing loss that occurs with aging and is found in 60% of those older than 65 years. It is a gradual sensorineural loss caused by nerve degeneration in the inner ear or auditory nerve, and it slowly progresses after the age of 50 years. The person first notices a high-frequency tone loss; it is harder to hear consonants (high-pitched components of speech) than vowels, which makes words sound garbled. The ability to localize sound is also impaired.

The nurse is assessing a patient who is obese for signs of metabolic syndrome. This condition is diagnosed when three or more certain risk factors are present. Which of these assessment findings are risk factors for metabolic syndrome? Select all that apply.

Answer: Fasting plasma glucose level greater than or equal to 110 mg/dL Blood pressure reading of 140/90 mm Hg Rationale: Metabolic syndrome is diagnosed when three or more of the following risk factors are present: (1) fasting plasma glucose level greater than or equal to 100 mg/dL; (2) blood pressure greater than or equal to 130/85 mm Hg; (3) waist circumference greater than or equal to 40 inches for men and 35 inches for women; (4) high-density lipoprotein cholesterol less than 40 in men and less than 50 in women; and (5) triglyceride levels greater than or equal to 150 mg/dL (ATP III, 2001).1. The nurse educator is preparing an education module for the nursing staff on the epidermal layer of skin. Which of these statements would be included in the module? The epidermis is:* Answer: Replaced every 4 weeks. Rationale: The epidermis is thin yet tough, replaced every 4 weeks, avascular, and stratified into several zones.

The nurse is assessing a patient who is admitted with possible delirium. Which of these are manifestations of delirium?

Answer: (SATA) Develops over a short period. Person is exhibiting memory impairment or deficits. Occurs as a result of a medical condition, such as systemic infection. Rationale: Delirium is a disturbance of consciousness that develops over a short period and may be attributable to a medical condition. Memory deficits may also occur. Apraxia and agnosia occur with dementia.

The nurse is evaluating patients for obesity-related diseases by calculating the waist-to-hip ratios. Which one of these patients would be at increased risk?

Answer: 29-year-old woman whose waist measures 33 inches and hips measure 36 inches Rationale: The waist-to-hip ratio assesses body fat distribution as an indicator of health risk. A waist-to-hip ratio of 1.0 or greater in men or 0.8 or greater in women is indicative of android (upper body obesity) and increasing risk for obesity-related disease and early death. The 29-year-old woman has a waist-to-hip ratio of 0.92, which is greater than 0.8. The 32-year-old man has a waist-to-hip ratio of 0.94; the 38-year-old man has a waist-to-hip ratio of 0.92; the 46-year-old woman has a waist-to-hip ratio of 0.78.

The nurse is conducting a class on alcohol and the effects of alcohol on the body. How many standard drinks (each containing 14 grams of alcohol) per day in men are associated with increased deaths from cirrhosis, cancers of the mouth, esophagus, and injuries?

Answer: 4 Rationale: In men, alcohol consumption of at least four standard drinks per day is associated with increased deaths from liver cirrhosis, cancers of the mouth, esophagus and other areas, and deaths from injuries and other external causes.

The nurse recognizes which of these persons is at greatest risk for undernutrition?

Answer: 5-month-old infant Rationale:Vulnerable groups for undernutrition are infants, children, pregnant women, recent immigrants, persons with low incomes, hospitalized people, and aging adults.

A father brings in his 2-month-old infant to the clinic because the infant has had diarrhea for the last 24 hours. He says his baby has not been able to keep any formula down and that the diarrhea has been at least every 2 hours. The nurse suspects dehydration. The nurse should test skin mobility and turgor over the infants:

Answer: Abdomen. Rationale: Mobility and turgor are tested over the abdomen in an infant. Poor turgor, or tenting, indicates dehydration or malnutrition. The other sites are not appropriate for checking skin turgor in an infant.

During an examination of the eye, the nurse would expect what normal finding when assessing the lacrimal apparatus?

Answer: Absence of drainage from the puncta when pressing against the inner orbital rim Rationale: No swelling, redness, or drainage from the puncta should be observed when it is pressed. Regurgitation of fluid from the puncta, when pressed, indicates duct blockage. The lacrimal glands are not functional at birth.

The nurse is providing care for a 68-year-old woman who is complaining of constipation. What concern exists regarding her nutritional status?

Answer: Absorption of nutrients may be impaired. Rationale: Gastrointestinal symptoms such as vomiting, diarrhea, or constipation may interfere with nutrient intake or absorption. The other responses are not correct.

The nurse is assessing a patient with a history of intravenous drug abuse. In assessing his mouth, the nurse notices a dark red confluent macule on the hard palate. This could be an early sign of:

Answer: Acquired immunodeficiency syndrome (AIDS). Rationale: Oral Kaposis sarcoma is a bruiselike, dark red or violet, confluent macule that usually occurs on the hard palate. It may appear on the soft palate or gingival margin. Oral lesions may be among the earliest lesions to develop with AIDS.

While performing the otoscopic examination of a 3-year-old boy who has been pulling on his left ear, the nurse finds that his left tympanic membrane is bright red and that the light reflex is not visible. The nurse interprets these findings to indicate a(n):

Answer: Acute otitis media. Rationale: Absent or distorted light reflex and a bright red color of the eardrum are indicative of acute otitis media.

How should the nurse perform a triceps skinfold assessment?

Answer: After applying the calipers, the nurse waits 3 seconds before taking a reading. After repeating the procedure three times, an average is recorded. Rationale: While holding the skinfold, the lever of the calipers is released. The nurse waits 3 seconds and then takes a reading. This procedure should be repeated three times, and an average of the three skinfold measurements is then recorded.

The nurse is assessing a patient who may have hearing loss. Which of these statements is true concerning air conduction?

Answer: Air conduction is the normal pathway for hearing. Rationale: The normal pathway of hearing is air conduction, which starts when sound waves produce vibrations on the tympanic membrane. Conductive hearing loss results from a mechanical dysfunction of the external or middle ear. The other statements are not true concerning air conduction.

The nurse is assessing the mental status of a child. Which statement about children and mental status is true?

Answer: All aspects of mental status in children are interdependent Rationale: Separating and tracing the development of only one aspect of mental status is difficult. All aspects are interdependent

When examining the nares of a 45-year-old patient who has complaints of rhinorrhea, itching of the nose and eyes, and sneezing, the nurse notices the following: pale turbinates, swelling of the turbinates, and clear rhinorrhea. Which of these conditions is most likely the cause?

Answer: Allergic rhinitis Rationale: Rhinorrhea, itching of the nose and eyes, and sneezing are present with allergic rhinitis. On physical examination, serous edema is noted, and the turbinates usually appear pale with a smooth, glistening surface.

The nurse is reviewing causes of increased intraocular pressure. Which of these factors determines intraocular pressure?

Answer: Amount of aqueous produced and resistance to its outflow at the angle of the anterior chamber Rationale: Intraocular pressure is determined by a balance between the amount of aqueous produced and the resistance to its outflow at the angle of the anterior chamber. The other responses are incorrect.

During an assessment of a 20-year-old patient with a 3-day history of nausea and vomiting, the nurse notices dry mucosa and deep vertical fissures in the tongue. These findings are reflective of:

Answer: Dehydration. Rationale: Dry mouth occurs with dehydration or fever. The tongue has deep vertical fissures.

The nurse is bathing an 80-year-old man and notices that his skin is wrinkled, thin, lax, and dry. This finding would be related to which factor in the older adult?

Answer: An increased loss of elastin and a decrease in subcutaneous fat Rationale: An accumulation of factors place the aging person at risk for skin disease and breakdown: the thinning of the skin, a decrease in vascularity and nutrients, the loss of protective cushioning of the subcutaneous layer, a lifetime of environmental trauma to skin, the social changes of aging, a increasingly sedentary lifestyle, and the chance of immobility.

25. An assessment of a 23-year-old patient reveals the following: an auricle that is tender and reddish-blue in color with small vesicles. The nurse would need to know additional information that includes which of these?

Answer: Any prolonged exposure to extreme cold Rationale: Frostbite causes reddish-blue discoloration and swelling of the auricle after exposure to extreme cold. Vesicles or bullae may develop, and the person feels pain and tenderness.

The nurse is performing an assessment on a 21-year-old patient and notices that his nasal mucosa appears pale, gray, and swollen. What would be the most appropriate question to ask the patient?

Answer: Are you aware of having any allergies? Rationale: With chronic allergies, the mucosa looks swollen, boggy, pale, and gray. Elevated body temperature, colds, and nosebleeds do not cause these mucosal changes.

A 30-year-old female patient is describing feelings of hopelessness and depression. She has attempted self-mutilation and has a history of suicide attempts. She describes difficulty sleeping at night and has lost 10 pounds in the past month. Which of these statements or questions is the nurses best response in this situation:

Answer: Are you feeling so hopeless that you feel like hurting yourself now? Rationale: When the person expresses feelings of hopelessness, despair, or grief, assessing the risk of physical harm to him or herself is important. This process begins with more general questions. If the answers are affirmative, then the assessment continues with more specific questions.

A 26 yr old woman was robbed and beaten a month ago. She is returning to the clinic today for a follow up assessment. The nurse will want to ask her which one of these questions?

Answer: Are you having any disturbing dreams? Rationale: In posttraumatic stress disorder, the person has been exposed to a traumatic event is persistently re experienced by recurrent and intrusive, distressing recollections of the event, including images, thoughts, or perceptions; recurrent distressing dreams of the event; and acting or feeling as if the traumatic event were recurring

The nurse notices the presence of periorbital edema when performing an eye assessment on a 70-year-old patient. The nurse should:

Answer: Ask the patient if he or she has a history of heart failure. Rationale: Periorbital edema occurs with local infections, crying, and systemic conditions such as heart failure, renal failure, allergy, and hypothyroidism. Periorbital edema is not associated with blepharitis.

During an assessment, the nurse notices that an older adult patient has tears rolling down his face from his left eye. Closer examination shows that the lower lid is loose and rolling outward. The patient complains of his eye feeling dry and itchy. Which action by the nurse is correct?

Answer: Assessing for other signs of ectropion Rationale: The condition described is known as ectropion, and it occurs in older adults and is attributable to atrophy of the elastic and fibrous tissues. The lower lid does not approximate to the eyeball, and, as a result, the puncta cannot effectively siphon tears; excessive tearing results. Ptosis is a drooping of the upper eyelid. These signs do not suggest the presence of a foreign body in the eye or basal cell carcinoma.

The nurse needs to pull the portion of the ear that consists of movable cartilage and skin down and back when administering eardrops. This portion of the ear is called the:

Answer: Auricle. Rationale: The external ear is called the auricle or pinna and consists of movable cartilage and skin.

The nurse is using an otoscope to assess the nasal cavity. Which of these techniques is correct?

Answer: Avoiding touching the nasal septum with the speculum Rationale: The correct technique for using an otoscope is to insert the apparatus into the nasal vestibule, avoiding pressure on the sensitive nasal septum. The tip of the nose should be lifted up before inserting the speculum.

A 52-year-old woman has a papule on her nose that has rounded, pearly borders and a central red ulcer. She said she first noticed it several months ago and that it has slowly grown larger. The nurse suspects which condition?

Answer: Basal cell carcinoma Rationale: Basal cell carcinoma usually starts as a skin-colored papule that develops rounded, pearly borders with a central red ulcer. It is the most common form of skin cancer and grows slowly. This description does not fit acne lesions.

The nurse is assessing a patient in the hospital who has received numerous antibiotics and notices that his tongue appears to be black and hairy. In response to his concern, what would the nurse say?

Answer: Black, hairy tongue is a fungal infection caused by all the antibiotics you have received. Rationalre: A black, hairy tongue is not really hair but the elongation of filiform papillae and painless overgrowth of mycelial threads of fungus infection on the tongue. It occurs after the use of antibiotics, which inhibit normal bacteria and allow a proliferation of fungus.

The nurse is assessing a 16-year-old patient who has suffered head injuries from a recent motor vehicle accident. Which of these statements indicates the most important reason for assessing for any drainage from the ear canal?

Answer: Bloody or clear watery drainage can indicate a basal skull fracture. Rationale: Frank blood or clear watery drainage (cerebrospinal leak) after a trauma suggests a basal skull fracture and warrants immediate referral. Purulent drainage indicates otitis externa or otitis media.

The nurse notices that the mother of a 2-year-old boy brings him into the clinic quite frequently for various injuries and suspects there may be some child abuse involved. During an inspection of his mouth, the nurse should look for:

Answer: Bruising on the buccal mucosa or gums. Rationale: The nurse should notice any bruising or laceration on the buccal mucosa or gums of an infant or young child. Trauma may indicate child abuse from a forced feeding of a bottle or spoon.

A mother asks when her newborn infants eyesight will be developed. The nurse should reply:

Answer: By approximately 3 months of age, infants develop more coordinated eye movements and can fixate on an object. Rationale: Eye movements may be poorly coordinated at birth, but by 3 to 4 months of age, the infant should establish binocularity and should be able to fixate simultaneously on a single image with both eyes.

A 40-year-old patient who has just finished chemotherapy for breast cancer tells the nurse that she is concerned about her mouth. During the assessment the nurse finds areas of buccal mucosa that are raw and red with some bleeding, as well as other areas that have a white, cheesy coating. The nurse recognizes that this abnormality is:

Answer: Candidiasis. Rationale: Candidiasis is a white, cheesy, curdlike patch on the buccal mucosa and tongue. It scrapes off, leaving a raw, red surface that easily bleeds. It also occurs after the use of antibiotics or corticosteroids and in persons who are immunosuppressed.

A semiconscious woman is brought to the emergency department after she was found on the floor in her kitchen. Her face, nail beds, lips, and oral mucosa are a bright cherry-red color. The nurse suspects that this coloring is due to:

Answer: Carbon monoxide poisoning. Rationale: A bright cherry-red coloring in the face, upper torso, nail beds, lips, and oral mucosa appears in cases of carbon monoxide poisoning.

A patient is especially worried about an area of skin on her feet that has turned white. The health care provider has told her that her condition is vitiligo. The nurse explains to her that vitiligo is:

Answer: Caused by the complete absence of melanin pigment Rationale: Vitiligo is the complete absence of melanin pigment in patchy areas of white or light skin on the face, neck, hands, feet, body folds, and around orifices, otherwise, the depigmented skin is normal.

During a nutritional assessment, why is it important for the nurse to ask a patient what medications he or she is taking?

Answer: Certain drugs can affect the metabolism of nutrients. Rationale: Analgesics, antacids, anticonvulsants, antibiotics, diuretics, laxatives, antineoplastic drugs, steroids, and oral contraceptives are drugs that can interact with nutrients, impairing their digestion, absorption, metabolism, or use. The other responses are not correct.

The nurse keeps in mind that a thorough skin assessment is extremely important because the skin holds information about a persons:

Answer: Circulatory status. Rationale: The skin holds information about the bodys circulation, nutritional status, and signs of systemic diseases, as well as topical data on the integumentary system itself.

A patient's repeatedly seems to have difficulty coming up with a word. He says, I was on my way to work, and when I got there, the thing that you step into that goes up in the air was so full that I decided to take the stairs. The nurse will note on his chart that he is using or experiencing:

Answer: Circumlocution Rationale: Circumlocution is a roundabout expression, substituting a phrase when one cannot think of the name of the object

A patient repeats, I feel hot. Hot, cot, rot, tot, got. Im a spot. The nurse documents this as an illustration of:

Answer: Clanging Rationale: Clanging is word choice based on sound, not meaning, and includes nonsense rhymes and puns.

A 65-year-old man with emphysema and bronchitis has come to the clinic for a follow-up appointment. On assessment, the nurse might expect to see which finding?

Answer: Clubbing of the nails Rationale: Clubbing of the nails occurs with congenital cyanotic heart disease and neoplastic and pulmonary diseases. The other responses are assessment findings not associated with pulmonary diseases.

A 70-year-old woman who loves to garden has small, flat, brown macules over her arms and hands. She asks, What causes these liver spots? The nurse tells her, They are:

Answer: Clusters of melanocytes that appear after extensive sun exposure. Rationale: Liver spots, or senile lentigines, are clusters of melanocytes that appear on the forearms and dorsa of the hands after extensive sun exposure. The other responses are not correct.

The nurse is administering a Mini-Cog test to an older adult woman. When asked to draw a clock showing the time of 10:45, the patient drew a clock with the numbers out of order and with an incorrect time. This result indicates which finding?

Answer: Cognitive impairment Rationale: The Mini-Cog is a newer instrument that screens for cognitive impairment, often found with dementia. The result of an abnormal drawing of a clock and time indicates a cognitive impairment.

A patient tells the nurse that he has noticed that one of his moles has started to burn and bleed. When assessing his skin, the nurse pays special attention to the danger signs for pigmented lesions and is concerned with which additional finding?

Answer: Color variation Rationale: Abnormal characteristics of pigmented lesions are summarized in the mnemonic ABCD: asymmetry of pigmented lesion, border irregularity, color variation, and diameter greater than 6 mm.

During an interview, the nurse notes that the patient gets up several times to wash her hands even though they are not dirty. This behavior is an example of:

Answer: Compulsive disorder Rationale: Repetitive behaviors, such as handwashing, are behaviors that the person feels driven to perform in response to an obsession. The behaviors are aimed at preventing or reducing distress or preventing some dreaded event or situation.

A patient with a middle ear infection asks the nurse, What does the middle ear do? The nurse responds by telling the patient that the middle ear functions to:

Answer: Conduct vibrations of sounds to the inner ear. Rationale: Among its other functions, the middle ear conducts sound vibrations from the outer ear to the central hearing apparatus in the inner ear. The other responses are not functions of the middle ear.

A woman has come to the clinic to seek help with a substance abuse problem. She admits to using cocaine just before arriving. Which of these assessment findings would the nurse expect to find when examining this woman?

Answer: Dilated pupils, pacing, and psychomotor agitation Rationale: A cocaine users appearance includes pupillary dilation, tachycardia or bradycardia, elevated or lowered blood pressure, sweating, chills, nausea, vomiting, and weight loss. The person's behavior includes euphoria, talkativeness, hypervigilance, pacing, psychomotor agitation, impaired social or occupational functioning, fighting, grandiosity, and visual or tactile hallucinations.

A physician has diagnosed a patient with purpura. After leaving the room, a nursing student asks the nurse what the physician saw that led to that diagnosis. The nurse should say, The physician is referring to the:

Answer: Confluent and extensive patch of petechiae and ecchymoses on the feet. Rationale: Purpura is a confluent and extensive patch of petechiae and ecchymoses and a flat macular hemorrhage observed in generalized disorders such as thrombocytopenia and scurvy. The blue dilation of blood vessels in a star-shaped linear pattern on the legs describes a venous lake. The fiery red, star-shaped marking on the cheek that has a solid circular center describes a spider or star angioma. The tiny areas of hemorrhage that are less than 2 mm, round, discrete, and dark red in color describes petechiae. A mother has noticed that her son, who has been to a new babysitter, has some blisters and scabs on his face and buttocks. On examination, the nurse notices moist, thin-roofed vesicles with a thin erythematous base and suspects:* Answer: Impetigo. Rationale: Impetigo is moist, thin-roofed vesicles with a thin erythematous base and is a contagious bacterial infection of the skin and most common in infants and children. Eczema is characterized by erythematous papules and vesicles with weeping, oozing, and crusts. Herpes zoster (i.e., chickenpox or varicella) is characterized by small, tight vesicles that are shiny with an erythematous base. Diaper dermatitis is characterized by red, moist maculopapular patches with poorly defined borders.

In using the ophthalmoscope to assess a patients eyes, the nurse notices a red glow in the patients pupils. On the basis of this finding, the nurse would:

Answer: Consider the red glow a normal reflection of the ophthalmoscope light off the inner retina. Rationale: The red glow filling the persons pupil is the red reflex and is a normal finding caused by the reflection of the ophthalmoscope light off the inner retina. The other responses are not correct.

A 2-week-old infant can fixate on an object but cannot follow a light or bright toy. The nurse would:

Answer: Consider this a normal finding. Rationale: By 2 to 4 weeks an infant can fixate on an object. By the age of 1 month, the infant should fixate and follow a bright light or toy.

When performing the corneal light reflex assessment, the nurse notes that the light is reflected at 2 oclock in each eye. The nurse should:

Answer: Consider this a normal finding. Rationale: Reflection of the light on the corneas should be in exactly the same spot on each eye, or symmetric. If asymmetry is noted, then the nurse should administer the cover test.

A patient has a normal pupillary light reflex. The nurse recognizes that this reflex indicates that:

Answer: Constriction of both pupils occurs in response to bright light. Rationale: The pupillary light reflex is the normal constriction of the pupils when bright light shines on the retina. The other responses are not correct.

The nurse educator is preparing an education module for the nursing staff on the dermis layer of skin. Which of these statements would be included in the module? The dermis:

Answer: Contains sensory receptors. Rationale: The dermis consists mostly of collagen, has resilient elastic tissue that allows the skin to stretch, and contains nerves, sensory receptors, blood vessels, and lymphatic vessels. It is not replaced every 4 weeks.

The nurse is assessing a patients eyes for the accommodation response and would expect to see which normal finding?

Answer: Convergence of the axes of the eyes Rationale: The accommodation reaction includes pupillary constriction and convergence of the axes of the eyes. The other responses are not correct.

A newborn infant has Down syndrome. During the skin assessment, the nurse notices a transient mottling in the trunk and extremities in response to the cool temperature in the examination room. The infants mother also notices the mottling and asks what it is. The nurse knows that this mottling is called:

Answer: Cutis marmorata. Rationale: Persistent or pronounced cutis marmorata occurs with infants born with Down syndrome or those born prematurely and is a transient mottling in the trunk and extremities in response to cool room temperatures. A cafe au lait spot is a large round or oval patch of light-brown pigmentation. Carotenemia produces a yellow-orange color in light-skinned persons. Acrocyanosis is a bluish color around the lips, hands and fingernails, and feet and toenails.

Which of these assessment findings would the nurse expect to see when examining the eyes of a black patient?

Answer: Dark retinal background Rationale: An ethnically based variability in the color of the iris and in retinal pigmentation exists, with darker irides having darker retinas behind them.

The nurse is assessing an 80-year-old patient. Which of these findings would be expected for this patient?

Answer: Decreased ability to identify odors Rationale: The sense of smell may be reduced because of a decrease in the number of olfactory nerve fibers. Nasal hairs grow coarser and stiffer with aging. The gums may recede with aging, not hypertrophy, and saliva production decreases.

When reviewing the use of alcohol by older adults, the nurse notes that older adults have several characteristics that can increase the risk of alcohol use. Which would increase the bioavailability of alcohol in the blood for longer periods in the older adult?

Answer: Decreased liver and kidney functioning Rationale: Decreased liver and kidney functioning increases the bioavailability of alcohol in the blood for longer periods. Aging people experience decreased muscle mass (not increased), which also increases the alcohol concentration in the blood because the alcohol is distributed to less tissue over time. Blood pressure and cardiac output are not factors regarding bioavailability.

A 21-year-old woman has been on a low-protein liquid diet for the past 2 months. She has had adequate intake of calories and appears well nourished. After further assessment, what would the nurse expect to find?

Answer: Decreased serum albumin Rationale: Kwashiorkor (protein malnutrition) is due to diets that may be high in calories but contain little or no protein (e.g., low-protein liquid diets, fad diets, and long-term use of dextrose-containing intravenous fluids). The serum albumin would be less than 3.5 g/dL.

Which of these interventions is most appropriate when the nurse is planning nutritional interventions for a healthy, active 74-year-old woman?

Answer: Decreasing the number of calories she is eating because of the decrease in energy requirements from the loss of lean body mass Rationale: Important nutritional features of the older years are a decrease in energy requirements as a result of loss of lean body mass, the most metabolically active tissue, and an increase in fat mass.

A patient comes to the clinic and tells the nurse that he has been confined to his recliner chair for approximately 3 days with his feet down and he asks the nurse to evaluate his feet. During the assessment, the nurse might expect to find:

Answer: Distended veins Rationale: Keeping the feet in a dependent position causes venous pooling, resulting in redness, warmth, and distended veins. Prolonged elevation would cause pallor and coolness. Immobilization or prolonged inactivity would cause prolonged capillary filling time.

While discussing the history of a 6-month-old infant, the mother tells the nurse that she took a significant amount of aspirin while she was pregnant. What question would the nurse want to include in the history?

Answer: Does your baby seem to startle with loud noises? Rationale: Children at risk for a hearing deficit include those exposed in utero to a variety of conditions, such as maternal rubella or to maternal ototoxic drugs.

An older adult woman is brought to the emergency department after being found lying on the kitchen floor for 2 days; she is extremely dehydrated. What would the nurse expect to see during the examination?

Answer: Dry mucous membranes and cracked lips Rationale: With dehydration, mucous membranes appear dry and the lips look parched and cracked. The other responses are not found in dehydration.

The nurse is preparing to measure fat and lean body mass and bone mineral density. Which tool is appropriate?

Answer: Dual-energy x-ray absorptiometry (DEXA) Rationale: DEXA measures both bone mineral density and fat and lean body mass. BIA measures fat and lean body mass but not bone mineral density. A measuring tape measures distance or length, and skinfold calipers are used to determine skinfold thickness.

A 92-year-old patient has had a stroke. The right side of his face is drooping. The nurse might also suspect which of these assessment findings?

Answer: Dysphagia Rationale: Dysphagia is difficulty with swallowing and may occur with a variety of disorders, including stroke and other neurologic diseases. Rhinorrhea is a runny nose, epistaxis is a bloody nose, and xerostomia is a dry mouth.

The nurse is examining a patient who tells the nurse, I sure sweat a lot, especially on my face and feet but it doesnt have an odor. The nurse knows that this condition could be related to:

Answer: Eccrine glands. Rationale: The eccrine glands are coiled tubules that directly open onto the skin surface and produce a dilute saline solution called sweat. Apocrine glands are primarily located in the axillae, anogenital area, nipples, and naval area and mix with bacterial flora to produce the characteristic musky body odor. The patients statement is not related to disorders of the stratum corneum or the stratum germinativum.

During morning rounds, the nurse asks a patient, How are you today? The patient responds, You today, you today, you today, and mumbles the words. This speech pattern is an example of:

Answer: Echolalia Rationale: Echolalia occurs when a person imitates or repeats anothers words or phrases, often with a mumbling, mocking, or a mechanical tone.

When examining a patients eyes, the nurse recalls that stimulation of the sympathetic branch of the autonomic nervous system:

Answer: Elevates the eyelid and dilates the pupil. Rationale: Stimulation of the sympathetic branch of the autonomic nervous system dilates the pupil and elevates the eyelid. Parasympathetic nervous system stimulation causes the pupil to constrict. The muscle fibers of the iris contract the pupil in bright light to accommodate for near vision. The ciliary body controls the thickness of the lens.

In an individual with otitis externa, which of these signs would the nurse expect to find on assessment?

Answer: Enlarged superficial cervical nodes Rationales: The lymphatic drainage of the external ear flows to the parotid, mastoid, and superficial cervical nodes. The signs are severe swelling of the canal, inflammation, and tenderness. Rhinorrhea, periorbital edema, and pain over the maxillary sinuses do not occur with otitis externa.

A patient comes to the clinic and states that he has noticed that his skin is redder than normal. The nurse understands that this condition is due to hyperemia and knows that it can be caused by:

Answer: Excess blood in the dilated superficial capillaries Rationale: Erythema is an intense redness of the skin caused by excess blood (hyperemia) in the dilated superficial capillaries.

The nurse is assessing the body weight as a percentage of ideal body weight on an adolescent patient who was admitted for suspected anorexia nervosa. The patients usual weight was 125 pounds, but today she weighs 98 pounds. The nurse calculates the patients ideal body weight and concludes that the patient is:

Answer: Experiencing moderate malnutrition. Rationale: By dividing her current weight by her usual weight and then multiplying by 100, a percentage of 78.4% is obtained, which means that her current weight is 78.4% of her ideal body weight. A current weight of 80% to 90% of ideal weight suggests mild malnutrition; a current weight of 70% to 80% of ideal weight suggests moderate malnutrition; a current weight of less than 70% of ideal weight suggests severe malnutrition.

The primary purpose of the ciliated mucous membrane in the nose is to:

Answer: Filter out dust and bacteria. Rationale: The nasal hairs filter the coarsest matter from inhaled air, whereas the mucous blanket filters out dust and bacteria. The rich blood supply of the nasal mucosa warms the inhaled air.

The nurse is discussing appropriate foods with the mother of a 3-year-old child. Which of these foods are recommended?

Answer: Finger foods and nutritious snacks that cannot cause choking Rationale: Small portions, finger foods, simple meals, and nutritious snacks help improve the dietary intake of young children. Foods likely to be aspirated should be avoided (e.g., hot dogs, nuts, grapes, round candies, popcorn).

16. The nurse is palpating the sinus areas. If the findings are normal, then the patient should report which sensation?

Answer: Firm pressure Rationale: The person should feel firm pressure but no pain. Sinus areas are tender to palpation in persons with chronic allergies or an acute infection (sinusitis).

A patient is asked to indicate on a form how many times he eats a specific food. This method describes which of these tools for obtaining dietary information?

Answer: Food-frequency questionnaire Rationale: With this tool, information is collected on how many times per day, week, or month the individual eats particular foods, which provides an estimate of usual intake.

The tissue that connects the tongue to the floor of the mouth is the:

Answer: Frenulum. Rationale: The frenulum is a midline fold of tissue that connects the tongue to the floor of the mouth. The uvula is the free projection hanging down from the middle of the soft palate. The palate is the arching roof of the mouth. Papillae are the rough, bumpy elevations on the tongues dorsal surface.

The nurse is planning to assess new memory with a patient. The best way for the nurse to do this would be to

Answer: Give him the Four Related Words Test Rationale: Ask questions that can be corroborated, which screens for the occasional person who confabulates or makes up answers to fill in the gaps of memory loss.

The nurse discovers speech problems in a patient during an assessment. The patient has spontaneous speech, but it is mostly absent or is reduced to a few stereotypical words or sounds. This finding reflects which type of aphasia?

Answer: Global Rationale: Global aphasia is the most common and severe form of aphasia. Spontaneous speech is absent or reduced to a few stereotyped words or sounds, and prognosis for language recovery is poor. Dysphonic aphasia is not a valid condition.

A patient describes feeling an unreasonable, irrational fear of snakes. His fear is so persistent that he can no longer comfortable look at even pictures of snakes and has made an effort to identify all the places he might encounter a snake and avoids them. The nurse recognizes that he:

Answer: Has a snake phobia Rationale: A phobia is a strong, persistent, irrational fear of an object or situation; the person feels driven to avoid it

A 72-year-old patient has a history of hypertension and chronic lung disease. An important question for the nurse to include in the health history would be:

Answer: Have you noticed any dryness in your mouth? Rationale: Xerostomia (dry mouth) is a side effect of many drugs taken by older people, including antidepressants, anticholinergics, antispasmodics, antihypertensives, antipsychotics, and bronchodilators.

A 16-year-old girl is being seen at the clinic for gastrointestinal complaints and weight loss. The nurse determines that many of her complaints may be related to erratic eating patterns, eating predominantly fast foods, and high caffeine intake. In this situation, which is most appropriate when collecting current dietary intake information?

Answer: Having the patient complete a food diary for 3 days, including 2 weekdays and 1 weekend day Rationale: Food diaries require the individual to write down everything consumed for a certain time period. Because of the erratic eating patterns of this individual, assessing dietary intake over a few days would produce more accurate information regarding eating patterns. Direct observation is best used with young children or older adults.

While performing an assessment of a 65-year-old man with a history of hypertension and coronary artery disease, the nurse notices the presence of bilateral pitting edema in the lower legs. The skin is puffy and tight but normal in color. No increased redness or tenderness is observed over his lower legs, and the peripheral pulses are equal and strong. In this situation, the nurse suspects that the likely cause of the edema is which condition?

Answer: Heart failure Rational: Bilateral edema or edema that is generalized over the entire body is caused by a central problem such as heart failure or kidney failure. Unilateral edema usually has a local or peripheral cause.

In teaching a patient how to determine total body fat at home, the nurse includes instructions to obtain measurements of:

Answer: Height and weight. Rationale: Body mass index, calculated by using height and weight measurements, is a practical marker of optimal weight for height and an indicator of obesity. The other options are not correct.

When considering a nutritional assessment, the nurse is aware that the most common anthropometric measurements include:

Answer: Height and weight. Rationale: The most commonly used anthropometric measures are height, weight, triceps skinfold thickness, elbow breadth, and arm and head circumferences.

The nurse is performing a mental status assessment on a 5 yr old girl. Her patients are undergoing a bitter divorce and are worried about the effect it is having on their daughter. Which action or statement might lead the nurse to be concerned about the girls mental status?

Answer: Her mother states that her daughter prefers to play with toddlers instead of kids her own age while in daycare Rationale: The mental status assessment of infants and children covers behavioral, cognitive, and psychosocial development and examines how the child is coping with his or her environment.

A patient is brought to the emergency department. He is restless, has dilated pupils, is sweating, has a runny nose and tearing eyes, and complains of muscle and joint pains. His girlfriend thinks he has influenza, but she became concerned when his temperature went up to 39.4 C. She admits that he has been a heavy drug user, but he has been trying to stop on his own. The nurse suspects that the patient is experiencing withdrawal symptoms from which substance?

Answer: Heroin Rationale: Withdrawal symptoms of opiates, such as heroin, are similar to the clinical picture of influenza and include symptoms such as dilated pupils, lacrimation, runny nose, tachycardia, fever, restlessness, muscle and joint pains, and other symptoms.

The nurse is performing an ear examination of an 80-year-old patient. Which of these findings would be considered normal?

Answer: High-tone frequency loss Rationale: A high-tone frequency hearing loss is apparent for those affected with presbycusis, the hearing loss that occurs with aging. The pinna loses elasticity, causing earlobes to be pendulous. The eardrum may be whiter in color and more opaque and duller in the older person than in the younger adult.

A patient comes into the clinic complaining of pain in her right eye. On examination, the nurse sees a pustule at the lid margin that is painful to touch, red, and swollen. The nurse recognizes that this is a:

Answer: Hordeolum (stye) Rationale: A hordeolum, or stye, is a painful, red, and swollen pustule at the lid margin. A chalazion is a nodule protruding on the lid, toward the inside, and is nontender, firm, with discrete swelling. Dacryocystitis is an inflammation of the lacrimal sac. Blepharitis is inflammation of the eyelids.

During a mental status exam, the nurse wants to assess a patient's affect. The nurse should ask the patient which question?

Answer: How do you feel today? Rationale: judge mood and affect by body language and facial expression and by directly asking, How do you feel today, or how do you usually feel?

A patient states, I feel so sad all of the time. I cant feel happy even doing things I used to like to do. He also states that he is tires, sleeps poorly, and has no energy. To differentiates b/w a dysthymic disorder and a major depressive disorder, the nurse should ask which question?

Answer: How long have you been feeling this way? Rationale: Major depressive disorder is characterized by one or more major depressives episode, that is, at least 2 weeks of depressed mood or loss of interest accompanied by at least 4 additional symptoms of depression.

The nurse is performing a nutritional assessment on a 15-year-old girl who tells the nurse that she is so fat. Assessment reveals that she is 5 feet 4 inches and weighs 110 pounds. The nurses appropriate response would be:

Answer: How much do you think you should weigh? Rationale: Adolescents increased body awareness and self-consciousness may cause eating disorders such as anorexia nervosa or bulimia, conditions in which the real or perceived body image does not favorably compare with an ideal image. The nurse should not belittle the adolescents feelings, provide unsolicited advice, or agree with her.

The nurse is assessing a patient who has been admitted for cirrhosis of the liver, secondary to chronic alcohol use. During the physical assessment, the nurse looks for cardiac problems that are associated with chronic use of alcohol, such as:

Answer: Hypertension. Rationale: Even moderate drinking leads to hypertension and cardiomyopathy, with an increase in left ventricular mass, dilation of ventricles, and wall thinning. Ventricular fibrillation, bradycardia, and mitral valve prolapse are not associated with chronic heavy use of alcohol.

During a physical education class, a student is hit in the eye with the end of a baseball bat. When examined in the emergency department, the nurse notices the presence of blood in the anterior chamber of the eye. This finding indicates the presence of:

Answer: Hyphema. Rationale: Hyphema is the term for blood in the anterior chamber and is a serious result of blunt trauma (a fist or a baseball) or spontaneous hemorrhage and may indicate scleral rupture or major intraocular trauma.

The nurse is assessing orientation in a 79 yr old patient. Which of these responses would lead the nurse to conclude that this patient is oriented?

Answer: I know my name is John. I am at a hospital in Spokane. I couldn't tell you what date it is, but I know that it is February of a new year 2010. Rationale: Many aging persons experience social isolation, loss of structure without a job, a change in residence, or some short-term memory loss

The nurse is conducting a patient interview. Which statement made by the patient should the nurse more fully explore during the interview?

Answer: I never did good in school Rationale: In every mental status examination the following factors from the health history that could affect the findings should be noted: known illness or health problems, current meds, pt. normal baseline vitals

A woman is leaving on a trip to Hawaii and has come in for a checkup. During the examination the nurse learns that she has diabetes and takes oral hypoglycemic agents. The patient needs to be concerned about which possible effect of her medications?

Answer: Importance of sunscreen and avoiding direct sunlight Rationale: Drugs that may increase sunlight sensitivity and give a burn response include sulfonamides, thiazide diuretics, oral hypoglycemic agents, and tetracycline.

A patient has been diagnosed with schizophrenia. During a recent interview, he shows the nurse a picture of a man holding a decapitated head. He describes this picture as horrifying but then laughs loudly at the content. This behavior is a display of

Answer: Inappropriate affect Rationale: An inappropriate affect is an effect clearly discordant with the content of the persons speech

37. An ophthalmic examination reveals papilledema. The nurse is aware that this finding indicates:

Answer: Increased intracranial pressure. Rationale: Papilledema, or choked disk, is a serious sign of increased intracranial pressure, which is caused by a spaceoccupying mass such as a brain tumor or hematoma. This pressure causes venous stasis in the globe, showing redness, congestion, and elevation of the optic disc, blurred margins, hemorrhages, and absent venous pulsations. Papilledema is not associated with the conditions in the other responses.

A woman who is in the second trimester of pregnancy mentions that she has had more nosebleeds than ever since she became pregnant. The nurse recognizes that this is a result of:

Answer: Increased vascularity in the upper respiratory tract as a result of the pregnancy. Rationale: Nasal stuffiness and epistaxis may occur during pregnancy as a result of increased vascularity in the upper respiratory tract. The nurse is teaching a health class to high-school boys. When discussing the topic of using smokeless tobacco (SLT), which of these statements are accurate? Select all that apply.* Answer: -Using SLT has been associated with a greater risk of oral cancer than smoking. -Pain is rarely an early sign of oral cancer. -Tooth decay is another risk of SLT because of the use of sugar as a sweetener. Rationale: One pinch of SLT in the mouth for 30 minutes delivers the equivalent of three cigarettes. Pain is rarely an early sign of oral cancer. Many brands of SLT are sweetened with sugars, which promotes tooth decay. SLT is not considered a healthy alternative to smoking, and the use of SLT has been associated with a greater risk of oral cancer than smoking.

A 31-year-old patient tells the nurse that he has noticed pain in his left ear when people speak loudly to him. The nurse knows that this finding:

Answer: Is a characteristic of recruitment. Rationale: Recruitment is significant hearing loss occurring when speech is at low intensity, but sound actually becomes painful when the speaker repeats at a louder volume. The other responses are not correct.

During an assessment of a 20-year-old Asian patient, the nurse notices that he has dry, flaky cerumen in his canal. What is the significance of this finding? This finding:

Answer: Is a normal finding, and no further follow-up is necessary. Rationale: Asians and Native Americans are more likely to have dry cerumen, whereas Blacks and Whites usually have wet cerumen.

A 13-year-old girl is interested in obtaining information about the cause of her acne. The nurse should share with her that acne:

Answer: Is caused by increased sebum production. Rationale: Approximately 90% of males and 80% of females will develop acne; causes are increased sebum production and epithelial cells that do not desquamate normally.

The nurse is taking the history of a patient who may have a perforated eardrum. What would be an important question in this situation?

Answer: Is there any relationship between the ear pain and the discharge you mentioned? Rationale: Typically with perforation, ear pain occurs first, stopping with a popping sensation, and then drainage occurs.

During a skin assessment, the nurse notices that a Mexican-American patient has skin that is yellowish-brown; however, the skin on the hard and soft palate is pink and the patients scleras are not yellow. From this finding, the nurse could probably rule out:

Answer: Jaundice Rationale: Jaundice is exhibited by a yellow color, which indicates rising levels of bilirubin in the blood. Jaundice is first noticed in the junction of the hard and soft palate in the mouth and in the scleras. A black patient is in the intensive care unit because of impending shock after an accident. The nurse expects to find what characteristics in this patients skin?* Answer: Ashen, gray, or dull. Rationale: Pallor attributable to shock, with decreased perfusion and vasoconstriction, in black-skinned people will cause the skin to appear ashen, gray, or dull.

A 52-year-old patient describes the presence of occasional floaters or spots moving in front of his eyes. The nurse should:

Answer: Know that floaters are usually insignificant and are caused by condensed vitreous fibers. Rationale: Floaters are a common sensation with myopia or after middle age and are attributable to condensed vitreous fibers. Floaters or spots are not usually significant, but the acute onset of floaters may occur with retinal detachment.

The nurse is performing a middle ear assessment on a 15-year-old patient who has had a history of chronic ear infections. When examining the right tympanic membrane, the nurse sees the presence of dense white patches. The tympanic membrane is otherwise unremarkable. It is pearly, with the light reflex at 5 oclock and landmarks visible. The nurse should:

Answer: Know that these are scars caused from frequent ear infections. Rationales: Dense white patches on the tympanic membrane are sequelae of repeated ear infections. They do not necessarily affect hearing.

A 20 yr old construction worker has been brought into the ER with heat stroke. He has delirium as a result of a fluid and electrolyte imbalance. For the mental status exam, the nurse should first assess the patient's:

Answer: LOC and cognitive abilities Rationale: Delirium is a disturbance of consciousness with reduced ability to focus, sustain, or shift attention. Delirium is not an alt. in mood, affect, or language ability

The nurse just noted from the medical record that the patient has a lesion that is confluent in nature. On examination, the nurse expects to find:

Answer: Lesions that run together. Rationale: Confluent lesions (as with urticaria [hives]) run together. Grouped lesions are clustered together. Annular lesions are circular in nature. Zosteriform lesions are arranged along a nerve route.

A patient drifts off to sleep when she is not being stimulated. The nurse can easily arouse her by calling her name, but the patient remains drowsy during the conversation. The best description of this patient's LOC would be:

Answer: Lethargic Rationale: Lethargic (or somnolet) is when the person is not fully alert, drifts off to sleep when not stimulated, and can be aroused when called by name in a normal voice but looks drowsy.

The nurse is performing an oral assessment on a 40-year-old Black patient and notices the presence of a 1 cm, non tender, grayish-white lesion on the left buccal mucosa. Which one of these statements is true? This lesion is:

Answer: Leukoedema and is common in dark-pigmented persons. Rationale: Leukoedema, a grayish-white benign lesion occurring on the buccal mucosa, is most often observed in Blacks.

A patient has had a terrible itch for several months that he has been continuously scratching. On examination, the nurse might expect to find:

Answer: Lichenification. Rationale: Lichenification results from prolonged, intense scratching that eventually thickens the skin and produces tightly packed sets of papules. A keloid is a hypertrophic scar. A fissure is a linear crack with abrupt edges, which extends into the dermis; it can be dry or moist. Keratoses are lesions that are raised, thickened areas of pigmentation that appear crusted, scaly, and warty.

The nurse is reviewing the nutritional assessment of an 82-year-old patient. Which of these factors will most likely affect the nutritional status of an older adult?

Answer: Living alone on a fixed income Rationale: Socioeconomic conditions frequently affect the nutritional status of the aging adult; these factors should be closely evaluated. Physical limitations, income, and social isolation are frequent problems that interfere with the acquisition of a balanced diet. A decrease in taste and smell and decreased gastrointestinal motility and absorption occur with aging. Cardiovascular status is not a factor that affects an older adults nutritional status.

The nurse is reviewing in age-related changes in the eye for a class. Which of these physiologic changes is responsible for presbyopia?

Answer: Loss of lens elasticity Rationale: The lens loses elasticity and decreases its ability to change shape to accommodate for near vision. This condition is called presbyopia.

A few days after a summer hiking trip, a 25-year-old man comes to the clinic with a rash. On examination, the nurse notes that the rash is red, macular, with a bulls eye pattern across his midriff and behind his knees. The nurse suspects:

Answer: Lyme disease. Rationale: Lyme disease occurs in people who spend time outdoors in May through September. The first disease state exhibits the distinctive bulls eye and a red macular or papular rash that radiates from the site of the tick bite with some central clearing. The rash spreads 5 cm or larger, and is usually in the axilla, midriff, inguinal, or behind the knee, with regional lymphadenopathy.

A 68-year-old woman is in the eye clinic for a checkup. She tells the nurse that she has been having trouble reading the paper, sewing, and even seeing the faces of her grandchildren. On examination, the nurse notes that she has some loss of central vision but her peripheral vision is normal. These findings suggest that she may have:

Answer: Macular degeneration. Rationale: Macular degeneration is the most common cause of blindness. It is characterized by the loss of central vision. Cataracts would show lens opacity. Chronic open-angle glaucoma, the most common type of glaucoma, involves a gradual loss of peripheral vision. These findings are not consistent with vision that is considered normal at any age.

The nurse is providing nutrition information to the mother of a 1-year-old child. Which of these statements represents accurate information for this age group?

Answer: Maintaining adequate fat and caloric intake is important for a child in this age group. Rationale: Because of rapid growth, especially of the brain, both infants and children younger than 2 years of age should not drink skim or low-fat milk or be placed on low-fat diets. Fats (calories and essential fatty acids) are required for proper growth and central nervous system development.

During reporting, the nurse hears that patient is experiencing hallucinations. Which is an example of a hallucination:

Answer: Man believes that his dead wife is talking to him Rationale: Hallucinations are sensory perceptions for which no external stimuli exists. They may strike any sense: visual, auditory, tactile, olfactory, or gustatory

Which of these conditions is due to an inadequate intake of both protein and calories?

Answer: Marasmus Rationale: Marasmus, protein-calorie malnutrition, is due to an inadequate intake of protein and calories or prolonged starvation. Obesity is due to caloric excess; bulimia is an eating disorder. Kwashiorkor is protein malnutrition.

During a session on substance abuse, the nurse is reviewing statistics with the class. For persons aged 12 years and older, which illicit substance was most commonly used?

Answer: Marijuana Rationale: In persons age 12 years and older who reported using during the past month, marijuana (hashish) was the most commonly used illicit drug reported.

The nurse is reviewing the development of the newborn infant. Regarding the sinuses, which statement is true in relation to a newborn infant?

Answer: Maxillary and ethmoid sinuses are the only sinuses present at birth. Rationale: Only the maxillary and ethmoid sinuses are present at birth. The sphenoid sinuses are minute at birth and develop after puberty. The frontal sinuses are absent at birth, are fairly well developed at age 7 to 8 years, and reach full size after puberty.

A patient comes into the clinic complaining of facial pain, fever, and malaise. On examination, the nurse notes swollen turbinates and purulent discharge from the nose. The patient also complains of a dull, throbbing pain in his cheeks and teeth on the right side and pain when the nurse palpates the areas. The nurse recognizes that this patient has:

Answer: Maxillary sinusitis. Rationale: Signs of maxillary sinusitis include facial pain after upper respiratory infection, red swollen nasal mucosa, swollen turbinates, and purulent discharge. The person also has fever, chills, and malaise. With maxillary sinusitis, dull throbbing pain occurs in the cheeks and teeth on the same side, and pain with palpation is present. With frontal sinusitis, pain is above the supraorbital ridge.

The nurse if assessing a 75 year old man. As the nurse begins the mental status portion of the assessments, the nurse expects that this patient?

Answer: May take a little longer to respond, but his general knowledge and abilities should not have declined Rationale:The aging process leaves the parameters of mental status most intactly. General knowledge does not decrease, and little or no loss in vocabulary occurs. Response time is slower that in a youth.

For the first time, the nurse is seeing a patient who has no history of nutrition-related problems. The initial nutritional screening should include which activity?

Answer: Measurement of weight and weight history Rationale: Parameters used for nutrition screening typically include weight and weight history, conditions associated with increased nutritional risk, diet information, and routine laboratory data. The other responses reflect a more indepth assessment rather than a screening.

During the aging process, the hair can look gray or white and begin to feel thin and fine. The nurse knows that this occurs because of a decrease in the number of functioning:

Answer: Melanocytes. Rationale: In the aging hair matrix, the number of functioning melanocytes decreases; as a result, the hair looks gray or white and feels thin and fine. The other options are not correct.

A 19 yr old women comes to the clinic at the insistence of her brother. She is wearing black combat boots and a black lace nightgown over the top of her clothes. Her hair is dyed pink with black streaks throughout. She has several pierced holes in her nared and ears is wearing an earring through her eyebrow and heavy black makeup. The nurse concludes that:

Answer: More information should be gathered to decide whether her dress is appropriate

The nurse is assessing for clubbing of the fingernails and expects to find:

Answer: Nail bases with an angle of 180 degrees or greater and nail bases that feel spongy. Rationale: The normal nail is firm at its base and has an angle of 160 degrees. In clubbing, the angle straightens to 180 degrees or greater and the nail base feels spongy.

A 70-year-old patient tells the nurse that he has noticed that he is having trouble hearing, especially in large groups. He says that he cant always tell where the sound is coming from and the words often sound mixed up. What might the nurse suspect as the cause for this change?

Answer: Nerve degeneration in the inner ear Rationale: Presbycusis is a type of hearing loss that occurs in 60% of those older than 65 years of age, even in those living in a quiet environment. This sensorineural loss is gradual and caused by nerve degeneration in the inner ear. Words sound garbled, and the ability to localize sound is also impaired. This communication dysfunction is accentuated when background noise is present.

A woman who has just discovered that she is pregnant is in the clinic for her first obstetric visit. She asks the nurse, How many drinks a day is safe for my baby? The nurses best response is:

Answer: No amount of alcohol has been determined to be safe during pregnancy. Rationale: No amount of alcohol has been determined to be safe for pregnant women. The potential adverse effects of alcohol use on the fetus are well known; women who are pregnant should be screened for alcohol use, and abstinence should be recommended.

In assessing the tonsils of a 30 year old, the nurse notices that they are involuted, granular in appearance, and appear to have deep crypts. What is correct response to these findings?

Answer: No response is needed; this appearance is normal for the tonsils. Rationale: The tonsils are the same color as the surrounding mucous membrane, although they look more granular and their surface shows deep crypts. Tonsillar tissue enlarges during childhood until puberty and then involutes.

What should the nurse do next?

Answer: Notify the patients health care provider. Rationale: Any sudden loss of hearing in one or both ears that is not associated with an upper respiratory infection needs to be reported at once to the patients health care provider. Hearing loss associated with trauma is often sudden. Irrigating the ear or removing cerumen is not appropriate at this time.

If a 29-year-old woman weighs 156 pounds, and the nurse determines her ideal body weight to be 120 pounds, then how would the nurse classify the woman's weight?

Answer: Obese Rationale: Obesity, as a result of caloric excess, refers to weight more than 20% above ideal body weight. For this patient, 20% of her ideal body weight would be 24 pounds, and greater than 20% of her body weight would be over 144 pounds. Therefore, having a weight of 156 pounds would be considered obese.

The nurse notices that the patient is experiencing:

Answer: Objective vertigo. Rationale: With objective vertigo, the patient feels like the room spins; with subjective vertigo, the person feels like he or she is spinning. Tinnitus is a sound that comes from within a person; it can be a ringing, crackling, or buzzing sound. It accompanies some hearing or ear disorders. Dizziness is not the same as true vertigo; the person who is dizzy may feel unsteady and lightheaded.

A 60-year-old man is at the clinic for an eye examination. The nurse suspects that he has ptosis of one eye. How should the nurse check for this?

Answer: Observe the distance between the palpebral fissures. Rationale: Ptosis is a drooping of the upper eyelid that would be apparent by observing the distance between the upper and lower eyelids. The confrontation test measures peripheral vision. Measuring near vision or the corneal light test does not check for ptosis.

During an examination, the nurse can assess mental status by which activity?

Answer: Observing the patient and inferring health or dysfunction Rationale: Mental status cannot be directly scrutinized like the characteristics of skin or heart sounds. Its functioning is inferred through an assessment of an individual behaviors, such as consciousness, language, mood and affect, and other aspects

Which of these individuals would the nurse considerer at highest risk for suicide attempt?

Answer: Older adult man who tells the nurse that he is going to join his wife in heaven tomorrow and plans to use a gun Rationale: When the person expresses feelings of sadness, hopelessness, despair, or grief, assessing any possible risks of physical harm to him or herself is important. A precise suicide plan to take place in the next 24 to 48 hours with use of a lethal method constitutes high risk.

The nurse is examining a patients retina with an ophthalmoscope. Which finding is considered normal?

Answer: Optic disc that is a yellow-orange color Rationale: The optic disc is located on the nasal side of the retina. Its color is a creamy yellow-orange to a pink, and the edges are distinct and sharply demarcated, not blurred. A pigmented crescent is black and is due to the accumulation of pigment in the choroid.

A 31-year-old patient tells the nurse that he has noticed a progressive loss in his hearing. He says that it does seem to help when people speak louder or if he turns up the volume of a television or radio. The most likely cause of his hearing loss is:

Answer: Otosclerosis. Rationale: Otosclerosis is a common cause of conductive hearing loss in young adults between the ages of 20 and 40 years. Presbycusis is a type of hearing loss that occurs with aging. Trauma and frequent ear infections are not a likely cause of his hearing loss.

The nurse is assessing for inflammation in a dark-skinned person. Which technique is the best?

Answer: Palpating the skin for edema and increased warmth Rationale: Because inflammation cannot be seen in dark-skinned persons, palpating the skin for increased warmth, for taut or tightly pulled surfaces that may be indicative of edema, and for a hardening of deep tissues or blood vessels is often necessary.

The nurse notices that a patient has a solid, elevated, circumscribed lesion that is less than 1 cm in diameter. When documenting this finding, the nurse reports this as a:

Answer: Papule. Rationale: A papule is something one can feel, is solid, elevated, circumscribed, less than 1 cm in diameter, and is due to superficial thickening in the epidermis. A bulla is larger than 1 cm, superficial, and thin walled. A wheal is superficial, raised, transient, erythematous, and irregular in shape attributable to edema. A nodule is solid, elevated, hard or soft, and larger than 1 cm.

The nurse is performing the diagnostic positions test. Normal findings would be which of these results?

Answer: Parallel movement of both eyes Rationale: A normal response for the diagnostic positions test is parallel tracking of the object with both eyes. Eye movement that is not parallel indicates a weakness of an extraocular muscle or dysfunction of the CN that innervates it.

The salivary gland that is the largest and located in the cheek in front of the ear is the _________ gland.

Answer: Parotid Rationale: The mouth contains three pairs of salivary glands. The largest, the parotid gland, lies within the cheeks in front of the ear extending from the zygomatic arch down to the angle of the jaw. The Stensens duct (not gland) drains the parotid gland onto the buccal mucosa opposite the second molar. The sublingual gland is located within the floor of the mouth under the tongue. The submandibular gland lies beneath the mandible at the angle of the jaw.

The nurse is conducting a child safety class for new mothers. Which factor places young children at risk for ear infections?

Answer: Passive cigarette smoke Rationales: Exposure to passive and gestational smoke is a risk factor for ear infections in infants and children.

When examining the ear with an otoscope, the nurse notes that the tympanic membrane should appear:

Answer: Pearly gray and slightly concave. Rationale: The tympanic membrane is a translucent membrane with a pearly gray color and a prominent cone of light in the anteroinferior quadrant, which is the reflection of the otoscope light. The tympanic membrane is oval and slightly concave, pulled in at its center by the malleus, which is one of the middle ear ossicles.

A woman brings her husband to the clinic for an examination. She is particularly worried because after a recent fall, he seems to have lost a great deal of his memory of recent events. Which statement reflects the nurse's best course of action?

Answer: Perform a complete mental status examination Rationale: Performing a complete mental status exam is necessary when any abnormality in affect or behavior is discovered or when family members are concerned about a person's behavioral changes.

The nurse is assessing a 3 year old for drainage from the nose. On assessment, a purulent drainage that has a very foul odor is noted from the left naris and no drainage is observed from the right naris. The child is afebrile with no other symptoms. What should the nurse do next?

Answer: Perform an otoscopic examination of the left nares. Rationale: Children are prone to put an object up the nose, producing unilateral purulent drainage with a foul odor. Because some risk for aspiration exists, removal should be prompt.

In performing an examination of a 3-year-old child with a suspected ear infection, the nurse would:

Answer: Perform the otoscopic examination at the end of the assessment. Rationale: In addition to its place in the complete examination, eardrum assessment is mandatory for any infant or child requiring care for an illness or fever. For the infant or young child, the timing of the otoscopic examination is best toward the end of the complete examination.

A patient comes in for a physical examination and complains of freezing to death while waiting for her examination. The nurse notes that her skin is pale and cool and attributes this finding to:

Answer: Peripheral vasoconstriction. Rationale: A chilly or air-conditioned environment causes vasoconstriction, which results in false pallor and coolness.

A 23-year-old patient in the clinic appears anxious. Her speech is rapid, and she is fidgety and in constant motion. Which of these questions or statements would be most appropriate for the nurse to use in this situation to assess attention span?

Answer: Pick up the pencil in your left hand, move it to your right hand, and place it on the table. Rationale: Attention span is evaluated by assessing the individuals ability to concentrate and complete a thought or task without wandering. Giving a series of directions to follow is one method used to assess attention span.

A patient has been admitted for severe psoriasis. The nurse expects to see what finding in the patients fingernails?

Answer: Pitting Rationale: Sharply defined pitting and crumbling of the nails, each with distal detachment characterize pitting nails and are associated with psoriasis. The nurse is preparing for a certification course in skin care and needs to be familiar with the various lesions that may be identified on assessment of the skin. Which of the following definitions are correct? Select all that apply.* Answer: -Petechiae: Tiny punctate hemorrhages, 1 to 3 mm, round and discrete, dark red, purple, or brown in color -Vesicle: Known as a friction blister -Nodule: Solid, elevated, and hard or soft growth that is larger than 1 cm Rationale: A pustule is an elevated, circumscribed lesion filled with turbid fluid (pus). A hypertrophic scar is a keloid. A bulla is larger than 1 cm and contains clear fluid. A papule is solid and elevated but measures less than 1 cm.

A patient is admitted to the unit after an automobile accident. The nurse begins the mental status exam and finds that the patient has dysarthric speech and is lethargic. The nurses best approach regarding this exam is to?

Answer: Plan to defer the rest of the mental status exam Rationale: During the mental status exam, the sequence of steps form a hierarchy in which the most basic functions (consciousness, language) are assessed first. The first steps must be accurately assessed to ensure validity of the steps that follow

The nurse is planning health teaching for a 65-year-old woman who has had a cerebrovascular accident (stroke) and has aphasia. Which of these questions is most important to use when assessing mental status in this patient?

Answer: Please point to articles in the room and parts of the body as I name them. Rationale: Additional tests for persons with aphasia include word comprehension (asking the individual to point to articles in the room or parts of the body), reading (asking the person to read available print), and writing (asking the person to make up and write a sentence).

When a light is directed across the iris of a patients eye from the temporal side, the nurse is assessing for:

Answer: Presence of shadows, which may indicate glaucoma. Rationale: The presence of shadows in the anterior chamber may be a sign of acute angle-closure glaucoma. The normal iris is flat and creates no shadows. This method is not correct for the assessment of dacryocystitis, conjunctivitis, or cataracts.

During an assessment of the sclera of a black patient, the nurse would consider which of these an expected finding?

Answer: Presence of small brown macules on the sclera Rationale: Normally in dark-skinned people, small brown macules may be observed in the sclera.

During an oral assessment of a 30-year-old Black patient, the nurse notices bluish lips and a dark line along the gingival margin. What action would the nurse perform in response to this finding?

Answer: Proceed with the assessment, knowing that this appearance is a normal finding. Rationale: Some Blacks may have bluish lips and a dark line on the gingival margin; this appearance is a normal finding.

While obtaining a health history from the mother of a 1-year-old child, the nurse notices that the baby has had a bottle in his mouth the entire time. The mother states, It makes a great pacifier. The best response by the nurse would be:

Answer: Prolonged use of a bottle can increase the risk for tooth decay and ear infections. Rationale: Prolonged bottle use during the day or when going to sleep places the infant at risk for tooth decay and middle ear infections.

When assessing a patients nutritional status, the nurse recalls that the best definition of optimal nutritional status is sufficient nutrients that:

Answer: Provide for daily body requirements and support increased metabolic demands. Rationale: Optimal nutritional status is achieved when sufficient nutrients are consumed to support day-to-day body needs and any increased metabolic demands resulting from growth, pregnancy, or illness.

A 50-year-old woman with elevated total cholesterol and triglyceride levels is visiting the clinic to find out about her laboratory results. What would be important for the nurse to include in patient teaching in relation to these tests?

Answer: Provide information regarding a diet low in saturated fat. Rationale: The patient with elevated cholesterol and triglyceride levels should be taught about eating a healthy diet that limits the intake of foods high in saturated fats or trans fats. Reducing dietary fats is part of the treatment for this condition. The other responses are not pertinent to her condition.

The nurse is preparing to do an otoscopic examination on a 2-year-old child. Which one of these reflects the correct procedure?

Answer: Pulling the pinna down Rationales: For an otoscopic examination on an infant or on a child under 3 years of age, the pinna is pulled down. The other responses are not part of the correct procedure.

The nurse is performing an otoscopic examination on an adult. Which of these actions is correct?

Answer: Pulling the pinna up and back before inserting the speculum Rationale: The pinna is pulled up and back on an adult or older child, which helps straighten the S-shape of the canal. Traction should not be released on the ear until the examination is completed and the otoscope is removed.

The nurse is testing a patients visual accommodation, which refers to which action?

Answer: Pupillary constriction when looking at a near object Rationale: The muscle fibers of the iris contract the pupil in bright light and accommodate for near vision, which also results in pupil constriction. The other responses are not correct.

In a patient who has anisocoria, the nurse would expect to observe:

Answer: Pupils of unequal size. Rationale: Unequal pupil size is termed anisocoria. It normally exists in 5% of the population but may also be indicative of central nervous system disease.

A 45 yr old woman is at the clinic for a mental status assessment. In giving her the Four Related Words Test, the nurse would be concerned if she could not. four related words .

Answer: Recall: after a 30-minute delay

21. Immediately after birth, the nurse is unable to suction the nares of a newborn. An attempt is made to pass a catheter through both nasal cavities with no success. What should the nurse do next?

Answer: Recognize that this situation requires immediate intervention. Rationale: Determining the patency of the nares in the immediate newborn period is essential because most newborns are obligate nose breathers. Nares blocked with amniotic fluid are gently suctioned with a bulb syringe. If obstruction is suspected, then a small lumen (5 to 10 Fr) catheter is passed down each naris to confirm patency. The inability to pass a catheter through the nasal cavity indicates choanal atresia, which requires immediate intervention.

The nurse notices that a school-aged child has bluish-white, red-based spots in her mouth that are elevated approximately 1 to 3 mm. What other signs would the nurse expect to find in this patient?

Answer: Red-purple, maculopapular, blotchy rash behind the ears and on the face Rationale: With measles (rubeola), the examiner assesses a red-purple, blotchy rash on the third or fourth day of illness that appears first behind the ears, spreads over the face, and then over the neck, trunk, arms, and legs. The rash appears coppery and does not blanch. The bluish-white, red-based spots in the mouth are known as Koplik spots.

A 40-year-old woman reports a change in mole size, accompanied by color changes, itching, burning, and bleeding over the past month. She has a dark complexion and has no family history of skin cancer, but she has had many blistering sunburns in the past. The nurse would:

Answer: Refer the patient because of the suggestion of melanoma on the basis of her symptoms. Rationale: The ABCD danger signs of melanoma are asymmetry, border irregularity, color variation, and diameter. In addition, individuals may report a change in size, the development of itching, burning, and bleeding, or a new-pigmented lesion. Any one of these signs raises the suggestion of melanoma and warrants immediate referral.

The nurse is reviewing aspects of substance abuse in preparation for a seminar. Which of these statements illustrates the concept of tolerance to an illicit substance? The person:

Answer: Requires an increased amount of the substance to produce the same effect. Rationale: The concept of tolerance to a substance indicates that the person requires an increased amount of the substance to produce the same effect. Abuse indicates that the person needs to use the substance daily to function, and the person is unable to stop using it. Dependence is an actual physiologic dependence on the substance. Withdrawal occurs when cessation of the substance produces a syndrome of physiologic symptoms.

A patient has been diagnosed with strep throat. The nurse is aware that without treatment, which complication may occur?

Answer: Rheumatic fever Rationale: Untreated strep throat may lead to rheumatic fever. When performing a health history, the patient should be asked whether his or her sore throat has been documented as streptococcal.

During an assessment of a patient who has been homeless for several years, the nurse notices that his tongue is magenta in color, which is an indication of a deficiency in what mineral and/or vitamin?

Answer: Riboflavin Rationale: Magenta tongue is a sign of riboflavin deficiency. In contrast, a pale tongue is probably attributable to iron deficiency. Vitamin D and calcium deficiencies cause osteomalacia in adults, and a vitamin C deficiency causes scorbutic gums.

A patient in her first trimester of pregnancy is diagnosed with rubella. Which of these statements is correct regarding the significance of this in relation to the infants hearing?

Answer: Rubella can damage the infants organ of Corti, which will impair hearing. Rationale: If maternal rubella infection occurs during the first trimester, then it can damage the organ of Corti and impair hearing.

A man has come in to the clinic for a skin assessment because he is worried he might have skin cancer. During the skin assessment the nurse notices several areas of pigmentation that look greasy, dark, and stuck on his skin. Which is the best prediction?

Answer: Seborrheic keratoses, which do not become cancerous Rationale: Seborrheic keratoses appear like dark, greasy, stuck-on lesions that primarily develop on the trunk. These lesions do not become cancerous. Senile lentigines are commonly called liver spots and are not precancerous. Actinic (senile or solar) keratoses are lesions that are red-tan scaly plaques that increase over the years to become raised and roughened. They may have a silvery-white scale adherent to the plaque. They occur on sun-exposed surfaces and are directly related to sun exposure. They are premalignant and may develop into squamous cell carcinoma. Acrochordons are skin tags and are not precancerous.

A 42-year-old woman complains that she has noticed several small, slightly raised, bright red dots on her chest. On examination, the nurse expects that the spots are probably:

Answer: Senile angiomas. Rationale: Cherry (senile) angiomas are small, smooth, slightly raised bright red dots that commonly appear on the trunk of adults over 30 years old.

When assessing aging adults, the nurse knows that one of the first things that should be assessed before making judgements about their mental status is?

Answer: Sensory-perceptive abilities Rationale: Age-related changes in sensory perception can affect mental status

A patient comes to the emergency department after a boxing match, and his left eye is swollen almost shut. He has bruises on his face and neck. He says he is worried because he cant see well from his left eye. The physician suspects retinal damage. The nurse recognizes that signs of retinal detachment include:

Answer: Shadow or diminished vision in one quadrant or one half of the visual field. Rationale: With retinal detachment, the person has shadows or diminished vision in one quadrant or one half of the visual field. The other responses are not signs of retinal detachment.

A patient comes into the emergency department after an accident at work. A machine blew dust into his eyes, and he was not wearing safety glasses. The nurse examines his corneas by shining a light from the side across the cornea. What findings would suggest that he has suffered a corneal abrasion?

Answer: Shattered look to the light rays reflecting off the cornea Rationale: A corneal abrasion causes irregular ridges in reflected light, which produce a shattered appearance to light rays. No opacities should be observed in the cornea. The other responses are not correct.

The nurse is obtaining a health history on a 3-month-old infant. During the interview, the mother states, I think she is getting her first tooth because she has started drooling a lot. The nurses best response would be:

Answer: She is just starting to salivate and hasnt learned to swallow the saliva. Rationale: In the infant, salivation starts at 3 months. The baby will drool for a few months before learning to swallow the saliva. This drooling does not herald the eruption of the first tooth, although many parents think it does.

When assessing the pupillary light reflex, the nurse should use which technique?

Answer: Shine a light across the pupil from the side, and observe for direct and consensual pupillary constriction. Rationale: To test the pupillary light reflex, the nurse should advance a light in from the side and note the direct and consensual pupillary constriction.

A patient is unable to read even the largest letters on the Snellen chart. The nurse should take which action next?

Answer: Shorten the distance between the patient and the chart until the letters are seen, and record that distance. Rationale: If the person is unable to see even the largest letters when standing 20 feet from the chart, then the nurse should shorten the distance to the chart until the letters are seen, and record that distance (e.g., 10/200). If visual acuity is even lower, then the nurse should assess whether the person can count fingers when they are spread in front of the eyes or can distinguish light perception from a penlight. If vision is poorer than 20/30, then a referral to an ophthalmologist or optometrist is necessary, but the nurse must first assess the visual acuity. A patient's vision is recorded as 20/80 in each eye. The nurse interprets this finding to mean that the patient:* Answer: Has poor vision. Rationale: Normal visual acuity is 20/20 in each eye; the larger the denominator, the poorer the vision.

While obtaining a health history, a patient tells the nurse that he has frequent nosebleeds and asks the best way to get them to stop. What would be the nurses best response?

Answer: Sit up with your head tilted forward and pinch your nose. Rationale: With a nosebleed, the person should sit up with the head tilted forward and pinch the nose between the thumb and forefinger for 5 to 15 minutes.

The nurse is performing a nutritional assessment on an 80-year-old patient. The nurse knows that physiologic changes can directly affect the nutritional status of the older adult and include:

Answer: Slowed gastrointestinal motility. Rationale: Normal physiologic changes in aging adults that affect nutritional status include slowed gastrointestinal motility, decreased gastrointestinal absorption, diminished olfactory and taste sensitivity, decreased saliva production, decreased visual acuity, and poor dentition.

When assessing the tongue of an adult, the nurse knows that an abnormal finding would be:

Answer: Smooth glossy dorsal surface. Rationale: The dorsal surface of the tongue is normally roughened from papillae. A thin white coating may be present. The ventral surface may show veins. Smooth, glossy areas may indicate atrophic glossitis.

A mother and her 13-year-old daughter express their concern related to the daughters recent weight gain and her increase in appetite. Which of these statements represents information the nurse should discuss with them?

Answer: Snacks should be high in protein, iron, and calcium. Rationale: After a period of slow growth in late childhood, adolescence is characterized by rapid physical growth and endocrine and hormonal changes. Caloric and protein requirements increase to meet this demand. Because of bone growth and increasing muscle mass (and, in girls, the onset of menarche), calcium and iron requirements also increase.

The nurse has completed an assessment on a patient who came to the clinic for a leg injury. As a result of the assessment, the nurse has determined that the patient has at-risk alcohol use. Which action by the nurse is most appropriate at this time?

Answer: State, You are drinking more than is medically safe. I strongly recommend that you quit drinking, and Im willing to help you. Rationale: If an assessment has determined that the patient has at-risk drinking behavior, then the nurse should give a short but clear statement of assistance and concern. Simply giving out a telephone number or referral to agencies may not be enough.

During ocular examinations, the nurse keeps in mind that movement of the extraocular muscles is:

Answer: Stimulated by CNs III, IV, and VI. Rationale: Movement of the extraocular muscles is stimulated by three CNs: III, IV, and VI. The nurse is performing an external eye examination. Which statement regarding the outer layer of the eye is true?* Answer: The outer layer of the eye is very sensitive to touch. Rationale: The cornea and the sclera make up the outer layer of the eye. The cornea is very sensitive to touch. The middle layer, the choroid, has dark pigmentation to prevent light from reflecting internally. The trigeminal nerve (CN V) and the facial nerve (CN VII) are stimulated when the outer surface of the eye is stimulated. The retina, in the inner layer of the eye, is where light waves are changed into nerve impulses.

A pregnant woman states that she is concerned about her gums because she has noticed they are swollen and have started bleeding. What would be an appropriate response by the nurse?

Answer: Swollen and bleeding gums can be caused by the change in hormonal balance in your system during pregnancy. Rationale: Gum margins are red and swollen and easily bleed with gingivitis. A changing hormonal balance may cause this condition to occur in pregnancy and puberty.

During an examination, the nurse notes that a patient is exhibiting flight of ideas. Which statement by the patient is an example of flight of ideas?

Answer: Take this pill? The pill is red. I see red. Red velvet is soft, soft as a baby's bottom. Rationale: Flight of ideas is demonstrated by an abrupt change, rapid skipping from topic to topic and practically continuous flow of accelerated speech.

During a mental status assessment, which question by the nurse would best assess a persons judgement?

Answer: Tell me what you plan to do once you are discharged from the hospital Rationale: A person exercises judgement when he or she can compare and evaluate the alternatives in a situation and reach an appropriates course of action

During an assessment of a 26 year old at the clinic for a spot on my lip I think is cancer, the nurse notices a group of clear vesicles with an erythematous base around them located at the lip-skin border. The patient mentions that she just returned from Hawaii. What would be the most appropriate response by the nurse?

Answer: Tell the patient that these vesicles are indicative of herpes simplex I or cold sores and that they will Rationale: Cold sores are groups of clear vesicles with a surrounding erythematous base. These evolve into pustules or crusts and heal in 4 to 10 days. The most likely site is the lip-skin junction. Infection often recurs in the same site. Recurrent herpes infections may be precipitated by sunlight, fever, colds, or allergy.

27. The nurse is assessing color vision of a male child. Which statement is correct? The nurse should:

Answer: Test for color vision once between the ages of 4 and 8 years. Rationale: Test boys only once for color vision between the ages of 4 and 8 years. Color vision is not tested in girls because it is rare in girls. Testing is performed with the Ishihara test, which is a series of polychromatic cards.

The nurse is performing an eye-screening clinic at a daycare center. When examining a 2-year-old child, the nurse suspects that the child has a lazy eye and should:

Answer: Test for strabismus by performing the corneal light reflex test. Rationale: Testing for strabismus is done by performing the corneal light reflex test and the cover test. The Snellen eye chart and confrontation test are not used to test for strabismus.

A 22-year-old woman comes to the clinic because of severe sunburn and states, I was out in the sun for just a couple of minutes. The nurse begins a medication review with her, paying special attention to which medication class?

Answer: Tetracyclines for acne Rationale: Drugs that may increase sunlight sensitivity and give a burn response include sulfonamides, thiazide diuretics, oral hypoglycemic agents, and tetracycline.

The nurse is preparing to conduct a mental status examination. Which statement is true regarding the mental status examination?

Answer: The full mental status examination is a systematic check of emotional and cognitive functioning.

The nurse is conducting a visual examination. Which of these statements regarding visual pathways and visual fields is true?

Answer: The image formed on the retina is upside down and reversed from its actual appearance in the outside world. Rationale: The image formed on the retina is upside down and reversed from its actual appearance in the outside world. The light rays are refracted through the transparent media of the eye before striking the retina, and the nerve impulses are conducted through the optic nerve tract to the visual cortex of the occipital lobe of the brain. The left side of the brain interprets vision for the right eye.

4. A newborn infant is in the clinic for a well-baby checkup. The nurse observes the infant for the possibility of fluid loss because of which of these factors?

Answer: The newborns skin is more permeable than that of the adult. Rationale: The newborns skin is thin, smooth, and elastic and is relatively more permeable than that of the adult; consequently, the infant is at greater risk for fluid loss. The subcutaneous layer in the infant is inefficient, not thick, and the sebaceous glands are present but decrease in size and production. Vernix caseosa is not produced after birth.

The nurse is preparing to perform an otoscopic examination of a newborn infant. Which statement is true regarding this examination?

Answer: The normal membrane may appear thick and opaque. Rationale: During the first few days after the birth, the tympanic membrane of a newborn often appears thickened and opaque. It may look injected and have a mild redness from increased vascularity. The other statements are not correct.

A patients vision is recorded as 20/30 when the Snellen eye chart is used. The nurse interprets these results to indicate that:

Answer: The patient can read at 20 feet what a person with normal vision can read at 30 feet. Rationale: The top number indicates the distance the person is standing from the chart; the denominator gives the distance at which a normal eye can see.

The nurse is examining a patients ears and notices cerumen in the external canal. Which of these statements about cerumen is correct?

Answer: The purpose of cerumen is to protect and lubricate the ear. Rationale: The ear is lined with glands that secrete cerumen, which is a yellow waxy material that lubricates and protects the ear.

The mother of a 2-year-old toddler is concerned about the upcoming placement of tympanostomy tubes in her sons ears. The nurse would include which of these statements in the teaching plan?

Answer: The purpose of the tubes is to decrease the pressure and allow for drainage. Rationale: Polyethylene tubes are surgically inserted into the eardrum to relieve middle ear pressure and to promote drainage of chronic or recurrent middle ear infections. Tubes spontaneously extrude in 6 months to 1 year.

A 75-year-old woman who has a history of diabetes and peripheral vascular disease has been trying to remove a corn on the bottom of her foot with a pair of scissors. The nurse will encourage her to stop trying to remove the corn with scissors because:

Answer: The woman could be at increased risk for infection and lesions because of her chronic disease. Rationale: A personal history of diabetes and peripheral vascular disease increases a persons risk for skin lesions in the feet or ankles. The patient needs to seek a professional for assistance with corn removal.

The nurse is preparing to assess the visual acuity of a 16-year-old patient. How should the nurse proceed?

Answer: Use the Snellen chart positioned 20 feet away from the patient. Rationale: The Snellen alphabet chart is the most commonly used and most accurate measure of visual acuity. The confrontation test is a gross measure of peripheral vision. The Jaeger card or newspaper tests are used to test near vision.

A 32-year-old woman is at the clinic for little white bumps in my mouth. During the assessment, the nurse notes that she has a 0.5 cm white, nontender papule under her tongue and one on the mucosa of her right cheek. What would the nurse tell the patient?

Answer: These bumps are Fordyce granules, which are sebaceous cysts and are not a serious condition. Rationale: Fordyce granules are small, isolated white or yellow papules on the mucosa of the cheek, tongue, and lips. These little sebaceous cysts are painless and are not significant. Chalky, white raised patches would indicate leukoplakia. In strep throat, the examiner would see tonsils that are bright red, swollen, and may have exudates or white spots.

A mother brings her 4-month-old infant to the clinic with concerns regarding a small pad in the middle of the upper lip that has been there since 1 month of age. The infant has no health problems. On physical examination, the nurse notices a 0.5-cm, fleshy, elevated area in the middle of the upper lip. No evidence of inflammation or drainage is observed. What would the nurse tell this mother?

Answer: This elevated area is a sucking tubercle caused from the friction of breastfeeding or bottle-feeding and is normal. Rationale: A normal finding in infants is the sucking tubercle, a small pad in the middle of the upper lip from the friction of breastfeeding or bottle-feeding. This condition is not caused by irritation, teething, or excessive drooling, and evaluation by another health care provider is not warranted.

The nurse is providing instructions to newly hired graduates for the minimental state examination (MMSE). Which statement best describes this examination?

Answer: This examination is a good tool to detect delirium and dementia and to differentiate these from psychiatric mental illness. Rationale: The MMSE is a quick, easy test of 11 questions and is used for initial and serial evaluations and can demonstrate a worsening or an improvement of cognition over time and with treatment. It evaluates cognitive functioning, not mood or thought processes. MMSE is a good screening tool to detect dementia and delirium and to differentiate these from psychiatric mental illness.

A mother is concerned because her 18-month-old toddler has 12 teeth. She is wondering if this is normal for a child of this age. The nurses best response would be:

Answer: This is a normal number of teeth for an 18 month old. Rationale: The guidelines for the number of teeth for children younger than 2 years old are as follows: the childs age in months minus the number 6 should be equal to the expected number of deciduous teeth. Normally, all 20 teeth are in by 2 years old. In this instance, the child is 18 months old, minus 6, equals 12 deciduous teeth expected.

A 45-year-old farmer comes in for a skin evaluation and complains of hair loss on his head. His hair seems to be breaking off in patches, and he notices some scaling on his head. The nurse begins the examination suspecting:

Answer: Tinea capitis. Rationale: Tinea capitis is rounded patchy hair loss on the scalp, leaving broken-off hairs, pustules, and scales on the skin, and is caused by a fungal infection. Lesions are fluorescent under a Wood light and are usually observed in children and farmers; tinea capitis is highly contagious.

When examining the mouth of an older patient, the nurse recognizes which finding is due to the aging process?

Answer: Tongue that looks smoother in appearance Rationale: In the aging adult, the tongue looks smoother because of papillary atrophy. The teeth are slightly yellowed and appear longer because of the recession of gingival margins.

A 10 year old is at the clinic for a sore throat that has lasted 6 days. Which of these findings would be consistent with an acute infection?

Answer: Tonsils 3+/1-4+ with large white spots Rationale: With an acute infection, tonsils are bright red and swollen and may have exudate or large white spots. Tonsils are enlarged to 2+, 3+, or 4+ with an acute infection.

A mother brings her child into the clinic for an examination of the scalp and hair. She states that the child has developed irregularly shaped patches with broken-off, stublike hair in some places; she is worried that this condition could be some form of premature baldness. The nurse tells her that it is:

Answer: Trichotillomania; her child probably has a habit of absentmindedly twirling her hair. Rationale: Trichotillomania, self-induced hair loss, is usually due to habit. It forms irregularly shaped patches with broken-off, stublike hairs of varying lengths. A person is never completely bald. It occurs as a child absentmindedly rubs or twirls the area while falling asleep, reading, or watching television.

The projections in the nasal cavity that increase the surface area are called the:

Answer: Turbinates. Rationale: The lateral walls of each nasal cavity contain three parallel bony projections: the superior, middle, and inferior turbinates. These increase the surface area, making more blood vessels and mucous membrane available to warm, humidify, and filter the inhaled air.

The nurse assesses the hearing of a 7-month-old by clapping hands. What is the expected response? The infant:

Answer: Turns his or her head to localize the sound. Rationale: With a loud sudden noise, the nurse should notice the infant turning his or her head to localize the sound and to respond to his or her own name. A startle reflex and acoustic blink reflex is expected in newborns; at age 3 to 4 months, the infant stops any movement and appears to listen.

The nurse is performing an assessment. Which of these findings would cause the greatest concern?

Answer: Ulceration on the side of the tongue with rolled edges Rationale: Ulceration on the side or base of the tongue or under the tongue raises the suspicion of cancer and must be investigated. The risk of early metastasis is present because of rich lymphatic drainage. The vesicle may be an aphthous ulcer, which is painful but not dangerous. The other responses are normal findings.

The nurse is performing an eye assessment on an 80-year-old patient. Which of these findings is considered abnormal?

Answer: Unequal pupillary constriction in response to light Rationale: Pupils are small in the older adult, and the pupillary light reflex may be slowed, but pupillary constriction should be symmetric. The assessment findings in the other responses are considered normal in older persons.

A 17-year-old student is a swimmer on her high schools swim team. She has had three bouts of otitis externa this season and wants to know what to do to prevent it. The nurse instructs her to:

Answer: Use rubbing alcohol or 2% acetic acid eardrops after every swim. Rationale: With otitis externa (swimmers ear), swimming causes the external canal to become waterlogged and swell; skinfolds are set up for infection. Otitis externa can be prevented by using rubbing alcohol or 2% acetic acid eardrops after every swim.

During a checkup, a 22-year-old woman tells the nurse that she uses an over-the-counter nasal spray because of her allergies. She also states that it does not work as well as it used to when she first started using it. The best response by the nurse would be:

Answer: Using these nasal medications irritates the lining of the nose and may cause rebound swelling. Rationale: The misuse of over-the-counter nasal medications irritates the mucosa, causing rebound swelling, which is a common problem.

The nurse is reviewing the function of the cranial nerves (CNs). Which CN is responsible for conducting nerve impulses to the brain from the organ of Corti?

Answer: VIII Rationale: The nerve impulses are conducted by the auditory portion of CN VIII to the brain.

An older adult patient in a nursing home has been receiving tube feedings for several months. During an oral examination, the nurse notes that patients gums are swollen, ulcerated, and bleeding in some areas. The nurse suspects that the patient has what condition?

Answer: Vitamin C deficiency Rationale: Vitamin C deficiency causes swollen, ulcerated, and bleeding gums, known as scorbutic gums. Rickets is a condition related to vitamin D and calcium deficiencies in infants and children. Linoleic-acid deficiency causes eczematous skin. Vitamin A deficiency causes Bitot spots and visual problems.

A 50-year-old patient has been brought to the emergency department after a housemate found that the patient could not get out of bed alone. He has lived in a group home for years but for several months has not participated in the activities and has stayed in his room. The nurse assesses for signs of undernutrition, and an x-ray study reveals that he has osteomalacia, which is a deficiency of:

Answer: Vitamin D and calcium. Rationale: Osteomalacia results from a deficiency of vitamin D and calcium in adults. Iron deficiency would result in anemia, riboflavin deficiency would result in magenta tongue, and vitamin C deficiency would result in scurvy.

A patient has had a cerebrovascular accident (stroke). He is trying very hard to communicate. He seems driven to speak and says, I buy obie get spirding and take my train. What is the best description of this patients problem?

Answer: Wernicke's aphasia Rationale: The person can hear sounds, and words but cannot related them to previous experiences. Often a great urge to speak is present.

A patient tells the nurse that his food simply does not have any taste anymore. The nurses best response would be:

Answer: When did you first notice this change? Rationale: With changes in appetite, taste, smell, or chewing or swallowing, the examiner should ask about the type of change and when the change occurred. These problems interfere with adequate nutrient intake. The other responses are not correct.

While performing an assessment of the mouth, the nurse notices that the patient has a 1-cm ulceration that is crusted with an elevated border and located on the outer third of the lower lip. What other information would be most important for the nurse to assess?

Answer: When the patient first noticed the lesion Rationale: With carcinoma, the initial lesion is round and indurated, but then it becomes crusted and ulcerated with an elevated border. Most cancers occur between the outer and middle thirds of the lip. Any lesion that is still unhealed after 2 weeks should be referred.

The nurse is asking an adolescent about illicit substance abuse. The adolescent answers, Yes, Ive used marijuana at parties with my friends. What is the next question the nurse should ask?

Answer: When was the last time you used marijuana? Rationale: If a patient admits to the use of illicit substances, then the nurse should ask, When was the last time you used drugs? and How much did you take that time? The other questions may be considered accusatory and are not conducive to gathering information.

In performing a voice test to assess hearing, which of these actions would the nurse perform?

Answer: Whisper a set of random numbers and letters, and then ask the patient to repeat them. Rationale: With the head 30 to 60 cm (1 to 2 feet) from the patients ear, the examiner exhales and slowly whispers a set of random numbers and letters, such as 5, B, 6. Normally, the patient is asked to repeat each number and letter correctly after hearing the examiner say them.

A patient has been in the intensive care unit for 10 days. He has just been moved to the med-surg unit, and the admitting nurse is planning to perform a mental status exam. During the tests of cognitive function, the nurse would expect that he:

Answer: Will be oriented to place and person, but the patient may not be certain of the date Rationale: the nurse can discern the orientation of cognitive function through the course of the interview or can directly and tactfully ask. Some people have trouble keeping up with the dates whiles in the hospital

During an examination, the nurse finds that a patient has excessive dryness of the skin. The best term to describe this condition is:

Answer: Xerosis. Rationale: Xerosis is the term used to describe skin that is excessively dry. Pruritus refers to itching, alopecia refers to hair loss, and seborrhea refers to oily skin.

During an otoscopic examination, the nurse notices an area of black and white dots on the tympanic membrane and the ear canal wall. What does this finding suggest?

Answer: Yeast or fungal infection Rationale: A colony of black or white dots on the drum or canal wall suggests a yeast or fungal infection (otomycosis).

The nurse is assessing a patient who has liver disease for jaundice. Which of these assessment findings is indicative of true jaundice?

Answer: Yellow color of the sclera that extends up to the iris Rationale: The yellow sclera of jaundice extends up to the edge of the iris. Calluses on the palms and soles of the feet often appear yellow but are not classified as jaundice. Scleral jaundice should not be confused with the normal yellow subconjunctival fatty deposits that are common in the outer sclera of dark-skinned persons.

The mother of a 2-year-old is concerned because her son has had three ear infections in the past year. What would be an appropriate response by the nurse?

Answer: Your sons eustachian tube is shorter and wider than yours because of his age, which allows for infections to develop more easily. Rationale: The infants eustachian tube is relatively shorter and wider than the adults eustachian tube, and its position is more horizontal; consequently, pathogens from the nasopharynx can more easily migrate through to the middle ear. The other responses are not appropriate.

A 35-year-old pregnant woman comes to the clinic for a monthly appointment. During the assessment, the nurse notices that she has a brown patch of hyper-pigmentation on her face. The nurse continues the skin assessment aware that another finding may be:

Answer:Chloasma. Rationale: In pregnancy, skin changes can include striae, linea nigra (a brownish-black line down the midline), chloasma (brown patches of hyper-pigmentation), and vascular spiders. Keratoses are raised, thickened areas of pigmentation that look crusted, scaly, and warty. Xerosis is dry skin. Acrochordons, or skin tags, occur more often in the aging adult.

During an assessment, the nurse asks a female patient, How many alcoholic drinks do you have a week? Which answer by the patient would indicate at-risk drinking?

Answer:I have seven or eight drinks a week, but I never get drunk. Rationale: For women, having seven or more drinks a week or three or more drinks per occasion is considered at-risk drinking, according to the National Institute on Alcohol Abuse and Alcoholism.

In performing an assessment on a 49-year-old woman who has imbalanced nutrition as a result of dysphagia, which data would the nurse expect to find?

Answer:Inadequate nutrient food intake Rationale: Dysphagia, or impaired swallowing, interferes with adequate nutrient intake.

During an oral examination of a 4-year-old Native-American child, the nurse notices that her uvula is partially split. Which of these statements is accurate?

Answers: A bifid uvula may occur in some Native-American groups. Rationale: Bifid uvula, a condition in which the uvula is split either completely or partially, occurs in some Native- American groups.

The nurse is performing an assessment on a 65-year-old man. He reports a crusty nodule behind the pinna. It intermittently bleeds and has not healed over the past 6 months. On physical assessment, the nurse finds an ulcerated crusted nodule with an indurated base. The preliminary analysis in this situation is that this:

Answers: Could be a potential carcinoma, and the patient should be referred for a biopsy. Rationale: An ulcerated crusted nodule with an indurated base that fails to heal is characteristic of a carcinoma. These lesions fail to heal and intermittently bleed. Individuals with such symptoms should be referred for a biopsy. The other responses are not correct.

The nurse is performing a mental status exam. Which statement is true regarding the assessment of mental status?

Answers: Mental Status functioning is inferred through the assessment of an individual's behaviors Rationale: Mental status functioning is inferred through the assessment of an individual's behaviors. It cannot be directly assessed like the characteristics of the skin or heart sounds.

When performing an otoscopic examination of a 5-year-old child with a history of chronic ear infections, the nurse sees that his right tympanic membrane is amber-yellow in color and that air bubbles are visible behind the tympanic membrane. The child reports occasional hearing loss and a popping sound with swallowing. The preliminary analysis based on this information is that the child:

Answers: Most likely has serous otitis media. Rationale: An amber-yellow color to the tympanic membrane suggests serum or pus in the middle ear. Air or fluid or bubbles behind the tympanic membrane are often visible. The patient may have feelings of fullness, transient hearing loss, and a popping sound with swallowing. These findings most likely suggest that the child has serous otitis media. The other responses are not correct.

The nurse is performing the Denver II screening on a 12 month old infant during a routine well-child visit. The nurse should tell the infant's parents that he Denver II:

Is a screening instrument designed to detect children who are slow in development

The nurse is reviewing the structures of the ear. Which of these statements concerning the eustachian tube is true?

Rationale: The eustachian tube allows an equalization of air pressure on each side of the tympanic membrane so that the membrane does not rupture during, for example, altitude changes in an airplane. The tube is normally closed, but it opens with swallowing or yawning.

The nurse is assessing a 30-year-old unemployed immigrant from an underdeveloped country who has been in the United States for 1 month. Which of these problems related to his nutritional status might the nurse expect to find?

c. Osteomalacia (softening of the bones) Rationale: General undernutrition, hypertension, diarrhea, lactose intolerance, osteomalacia, scurvy, and dental caries are among the more common nutrition-related problems of new immigrants from developing countries.


संबंधित स्टडी सेट्स

ECON 102: Chapter 16 - The Monetary System.

View Set

Chapter 2 Life Insurance Basic Part I

View Set

Basic Insurance Concepts and Priciples - Chapter Quiz

View Set

Week 8 Quiz Ch. 44, 45, 47, 51, 52

View Set

Chapter 35 Bowel Elimination PrepU

View Set

Which bone articulates with what?

View Set

US History - Unit 3 The Gilded Age Part 2

View Set

Poli-Sci (interest groups: Organizing for Influence

View Set